• YOU can help the next generation of students in the community!
    Share your trial papers and notes on our Notes & Resources page

Proving Question (2 Viewers)

seanieg89

Well-Known Member
Joined
Aug 8, 2006
Messages
2,662
Gender
Male
HSC
2007
An amusing corollary of FLT is that the n-th root of 2 is irrational for n > 2.

For if (a/b)^n = 2 for some integers a,b, then a^n = b^n + b^n provides a solution to the Fermat equation ;).

Sadly, Fermat's last theorem is not strong enough to prove that the SQUARE root of 2 is irrational.
 

Users Who Are Viewing This Thread (Users: 0, Guests: 2)

Top